Partial Differentiation -- If w=x+y and s=(x^3)+xy+(y^3), find 𝝏w/𝝏s

In summary: The formulae are just a shorthand for that observation.In summary, when finding a partial derivative with respect to a certain variable, it is important to specify what other variable is being held constant. This can be done by using the notation ##\partial f /\partial u|_v##, which represents the change in f as u varies while v stays constant. In some cases, it may be easier to differentiate normally, but treating the constant variable as a constant. Graphically, the partial derivative can be viewed as the component of the vector in the direction of the other variable.
  • #1
Robin64
34
3
Homework Statement
If w=x+y and s=(x^3)+xy+(y^3), find 𝝏w/𝝏s
Relevant Equations
𝝏w/𝝏s=(𝝏w/𝝏x)*(𝝏x/𝝏s)+(𝝏w/𝝏y)*(𝝏y/𝝏s)
𝝏w/𝝏x=1

and then I wasn't sure about 𝝏x/𝝏s, so I tried implicitly differentiating s:

1=(3x^2)(𝝏x/𝝏s)+y(𝝏x/𝝏s)+x(𝝏y/𝝏s)+(3y^2)(𝝏y/𝝏s)

And then I shaved my head in frustration.
 
  • Like
Likes Delta2
Physics news on Phys.org
  • #2
If you are taking a partial derivative, what parameter is being held constant?
 
  • #3
Is this the whole question, word for word? It doesn't make sense to me.
If s varies it is because x and or y varies, but a given change in s can be achieved by changes in x and y in different ways, and these lead to different changes in w.
 
  • Like
Likes Delta2 and Charles Link
  • #4
I forgot this part of the question. Apologies: (x^2)*y+x(y^2)=t
 
  • #5
Is ##dt=0 ##? i.e. is the partial derivative w.r.t. ## s ## at fixed ## t ##? You need to be more clear please.
 
  • Like
Likes Delta2
  • #6
This is all I know.
 

Attachments

  • Screen Shot 2020-11-06 at 22.10.37.png
    Screen Shot 2020-11-06 at 22.10.37.png
    5.2 KB · Views: 268
  • #7
Robin64 said:
This is all I know.
It could have been made clearer in the question, but from the full text one can guess that the partial wrt s means t constant, and mutatis mutandis.
 
  • Like
Likes Charles Link
  • #8
I apologize for that. Nothing else has flummoxed in calc, but whatever reason, this has done just that.
 
  • #9
Robin64 said:
I apologize for that. Nothing else has flummoxed in calc, but whatever reason, this has done just that.
Are you ok to go ahead now?
 
  • #10
Not really.
 
  • #11
Robin64 said:
Not really.
What equation do you get if you ##\partial/\partial s## the equation for t?
 
  • Like
Likes Charles Link
  • #12
Guys i don't understand why you interfere with ##t##. You can evaluate ##\frac{\partial x}{\partial s}## directly as $$\frac{\partial x}{\partial s}=\frac{1}{\frac{\partial s}{\partial x}}$$ you ll just find it as function of x and y instead of s and t.
Same logic applies to ##\frac{\partial y}{\partial s}##.
 
  • #13
t is just t(x,y). I don't see how 𝝏t/𝝏s help me.
 
  • #14
And frankly I don't see how to calculate 𝝏t/𝝏s
 
  • #15
Delta2 said:
Guys i don't understand why you interfere with ##t##. You can evaluate ##\frac{\partial x}{\partial s}## directly as $$\frac{\partial x}{\partial s}=\frac{1}{\frac{\partial s}{\partial x}}$$ you ll just find it as function of x and y instead of s and t.
Same logic applies to ##\frac{\partial y}{\partial s}##.
You can't do that with partials.
x=r cos(ΞΈ)
##\partial x/\partial r=\cos(\theta)##
##r^2=x^2+y^2##
##\partial r/\partial x=2x##
The flaw is that something different is being held constant in the two cases.
 
  • #16
haruspex said:
What equation do you get if you ##\partial/\partial s## the equation for t?
alternatively (for the OP) you can set ## dt=0 ##.
 
  • #17
haruspex said:
You can't do that with partials.
x=r cos(ΞΈ)
##\partial x/\partial r=\cos(\theta)##
##r^2=x^2+y^2##
##\partial r/\partial x=2x##
The flaw is that something different is being held constant in the two cases.
Yes you can. You have a mistake in the last line, you get that $$2r\frac{\partial r}{\partial x}=2x\Rightarrow\frac{\partial r}{\partial x}=\frac{x}{r}=\cos\theta$$
 
  • #18
Oh sorry , should have prove that ##\frac{\partial r}{\partial x}=\frac{1}{\cos\theta}##, maybe you are right @haruspex .
 
  • #19
Charles Link said:
alternatively (for the OP) you can set ## dt=0 ##.
When you say let dt=0, let the partial of t with respect to x plus the partial of t with respect to y=0?
 
  • #20
Delta2 said:
Yes you can. You have a mistake in the last line, you get that $$2r\frac{\partial r}{\partial x}=2x\Rightarrow\frac{\partial r}{\partial x}=\frac{x}{r}=\cos\theta$$
Yes, that was careless, but as you saw the corrected version makes my point.
 
  • #21
Charles Link said:
alternatively (for the OP) you can set ## dt=0 ##.
So if dt=0 then:

(2xy+y^2)dx+(2xy+x^2)dy=0...?
 
  • #22
Robin64 said:
When you say let dt=0, let the partial of t with respect to x plus the partial of t with respect to y=0?
No, @Charles Link and I are saying the same thing in different ways. Differentiate the expression for t partially wrt s.
The key is that if there are two independent variables u and v then ##\frac{\partial f}{\partial u}## is shorthand for ##\frac{\partial f}{\partial u}|_{v}##, i.e. the change in f as u varies but v stays constant. In particular, ##\frac{\partial v}{\partial u}=0## by definition.

In my Cartesian/polar example, the partial derivative wrt x implicitly means holding y constant, whereas the partial wrt r means holding theta constant.
 
  • Like
Likes Astronuc and Charles Link
  • #23
t isn't a function of s.
 
  • #24
Robin64 said:
t isn't a function of s.
No matter.. ##\partial t/\partial s## means the change in t if s changes but t doesn't, i.e. 0.
It produces what you got in post #21 except that, usefully, you have ##\partial x/\partial s## in place of dx and ##\partial y/\partial s## in place of dy.
 
  • #25
Ok, I see that.
 
  • #26
then I solve for 𝝏x/𝝏s and 𝝏y/𝝏s?
 
  • #27
So @haruspex, under suitable assumptions on what is being held constant (in your example if we keep ##\theta## constant) can we use the equation $$\frac{\partial x}{\partial s}=\frac{1}{\frac{\partial s}{\partial x}}$$
 
  • #28
Adding to my confusion is this. The solution to the problem is below, and I have no idea how to get there. Can anyone provide more illumination? I looked for resources that describe the application of the chain rule to these types of partial derivatives, but I can find nothing. Boas' "Mathematical Methods in the Physical Sciences" is less than helpful. Is there a YouTube video or a book that better describes how to approach a problem such as this one?
Screen Shot 2020-11-07 at 00.53.03.jpg
 
  • #29
Robin64 said:
then I solve for 𝝏x/𝝏s and 𝝏y/𝝏s?
In principle, you can differentiate both the expression for t and the expression for s partially wrt s to get two equations involving 𝝏x/𝝏s and 𝝏y/𝝏s, and hence solve for those. But it will get very messy.

A much easier way here is to rewrite the expression for s in terms of w and t, eliminating x and y. It takes a little ingenuity, but not much work.
 
  • Like
Likes Delta2 and Charles Link
  • #30
Delta2 said:
So @haruspex, under suitable assumptions on what is being held constant (in your example if we keep ##\theta## constant) can we use the equation $$\frac{\partial x}{\partial s}=\frac{1}{\frac{\partial s}{\partial x}}$$
If you elect to keep the same variable constant in both derivatives then you don't need partials at all. Just differentiate normally, but treating that variable as a constant.
E.g. if we set θ to be constant and write y = x tan(θ), we have 𝝏y/𝝏x = dy/dx = tan(θ) and 𝝏x/𝝏y = dx/dy = cot(θ).

Btw, the Cartesian/polar example can be viewed graphically. Draw a vector r and a corresponding vector x on the x axis. The partial derivative of one wrt the other is represented by the component of the one in the direction of the other. You can see that in each case you multiply by cos theta.
 
  • Like
Likes Delta2
  • #31
haruspex said:
In principle, you can differentiate both the expression for t and the expression for s partially wrt s to get two equations involving 𝝏x/𝝏s and 𝝏y/𝝏s, and hence solve for those. But it will get very messy.
I did it this way to verify the answer of post 28, and it's not too difficult if you use Kramer's rule type solution to the two linear equations.
 
  • Like
Likes Delta2
  • #32
So here in this problem when we take the partial derivatives ##\frac{\partial x}{\partial s},\frac{\partial y}{\partial s}## we assume that we keep t constant?
And when calculating ##\frac{\partial x}{\partial t},\frac{\partial y}{\partial t}## we assume that we keep s constant?
 
  • Like
Likes Charles Link
  • #33
Delta2 said:
So here in this problem when we take the partial derivatives ##\frac{\partial x}{\partial s},\frac{\partial y}{\partial s}## we assume that we keep t constant?
And when calculating ##\frac{\partial x}{\partial t},\frac{\partial y}{\partial t}## we assume that we keep s constant?
Yes. As I wrote, this could have been made clearer, but it is fairly standard that the reader is expected to figure out how the different sets of variables relate, and hence what is being kept constant in each partial derivative.
 
  • Like
Likes Charles Link and Delta2
  • #34
haruspex said:
Yes. As I wrote, this could have been made clearer, but it is fairly standard that the reader is expected to figure out how the different sets of variables relate, and hence what is being kept constant in each partial derivative.
Ok fine but something doesn't look quite right to me. The chain rule is
$$1=\frac{\partial s}{\partial s}=\frac{\partial s}{\partial x}\frac{\partial x}{\partial s}+\frac{\partial s}{\partial y}\frac{\partial y}{\partial s}$$.
In the above when we calculate for example ##\frac{\partial s}{\partial x}## we keep constant y, but when we calculate ##\frac{\partial x}{\partial s}## we keep constant t. Isn't that contradicting or it's that how the chain rule is supposed to work?
 
  • #35
Delta2 said:
Ok fine but something doesn't look quite right to me. The chain rule is
$$1=\frac{\partial s}{\partial s}=\frac{\partial s}{\partial x}\frac{\partial x}{\partial s}+\frac{\partial s}{\partial y}\frac{\partial y}{\partial s}$$.
In the above when we calculate for example ##\frac{\partial s}{\partial x}## we keep constant y, but when we calculate ##\frac{\partial x}{\partial s}## we keep constant t. Isn't that contradicting or it's that how the chain rule is supposed to work?
If you take x and y there as Cartesian coordinates and s as the r of polar coordinates your equation gives ##1=\cos^2(\theta)+\sin^2(\theta)##.
How about that?
 

Similar threads

  • Calculus and Beyond Homework Help
Replies
18
Views
1K
  • Calculus and Beyond Homework Help
Replies
6
Views
861
  • Calculus and Beyond Homework Help
Replies
2
Views
550
  • Calculus and Beyond Homework Help
Replies
6
Views
552
  • Calculus and Beyond Homework Help
Replies
25
Views
362
  • Calculus and Beyond Homework Help
Replies
10
Views
386
  • Calculus and Beyond Homework Help
Replies
5
Views
553
  • Calculus and Beyond Homework Help
Replies
7
Views
796
  • Calculus and Beyond Homework Help
Replies
4
Views
699
  • Calculus and Beyond Homework Help
Replies
6
Views
1K
Back
Top